math400-exercises-chapt0-co
math400-exercises-chapt0-co
Dr H. Gebran
Solution. a) We have to prove that every finite subset of {fn , n ∈ N∗ } is linearly independent.
But every finite subset of this set is contained in a set of the form P{f
N
1 , f2 , . . . , fN }. So we have
to prove that this set is linearly independent. Therefore, let k=1 αk fk = 0. This means
that N
P
α
k=1 k sin(kπx) = 0 ∀x ∈ [0, 1]. Let m ∈ {1, . . . , N }. Multiplying by sin(mπx) and
integrating we get
XN Z 1
αk sin(kπx) sin(mπx) dx = 0.
k=1 0
1
But using the trigonometric formula sin(kπx) sin(mπx) = 2 (cos(k − m)πx − cos(k + m)πx),
we see that (
Z 1 1
if k = m
sin(kπx) sin(mπx) dx = 2
0 0 ̸ m.
if k =
It follows that αm = 0. Since m was an arbitrary integer between 1 and N , we have αk =
0 ∀k = 1, . . . , N . This proves the linear independence of {f1 , f2 , . . . , fN } and therefore of the
infinite set {fn , n ∈ N∗ }.
PN kx = 0∀x ∈ [0, 1]. Consider the polynomial P (t) =
PN k
b) Suppose that k=1 αk e k=1 αk t .
x
Then, P (e ) = 0 ∀x ∈ [0, 1]. Therefore, P (y) = 0 ∀y ∈ [1, e]. Therefore, P has infinitely many
roots and so all its coefficients are zero.
2. Show that a linear map L : E → F between vector spaces is determined by its values on a basis.
Solution. Let B = {ei | i ∈ I} be a basis of E. Suppose that vi = Lei is given for all
i ∈ I.
P Now if x ∈ E, there exists a unique finite collection (αi )i∈J of real numbers such that
x = i∈J αj ej . We set
X
Lx = αj vj .
i∈J
It is easy to check that L is a linear operator.
3. Let E be an infinite dimensional normed space. Show that there is a linear functional L : E → R
which is not bounded. Hint: Show first that there is a basis {ei |i ∈ I} such that ||ei || = 1 ∀i ∈ I.
Since I is infinite, it contains an infinite countable subset J; let φ : N∗ → J be a bijection. Set
Leφ(n) = n and Lei = 0 for i ∈ / J.
1
Now set Leφ(n) = n and Lei = 0 (or any other value) if i ∈ / φ(N∗ ). By the previous exercise,
this defines a linear functional L : E → R. L is however unbounded because L is not bounded
on the unit ball (since eφ(n) ∈ BE and (Leφ(n) ) is unbounded).
4. Let E be a vector space and let || · ||1 and || · ||2 be two norms on E. We say that || · ||2 is
stronger than || · ||1 if there is a positive constant C such that ||x||1 ≤ C||x||2 for all x ∈ E.
Solution. a) Let B1 (x, r) = {y ∈ E | ||y − x||1 < r} and B2 (x, r) = {y ∈ E | ||y − x||2 < r}.
The inequality ||x||1 ≤ C||x||2 implies that B2 (x, Cr ) ⊂ B1 (x, r).
Now let O be open for || · ||1 and let x ∈ O. Then there exists r > 0 such that B1 (x, r) ⊂ O.
By the previous remark, B2 (x, Cr ) ⊂ O. Therefore O is open for || · ∥2 .
b) Observe first that ||u||1 ≤ ||u||∞ . On the other hand consider the sequence (un ) defined by
(
−n2 t + n if 0 ≤ t ≤ n1
un (t) =
0 otherwise.
Draw a figure. Then ||un ||1 = 1 but ||un ||∞ = n → ∞ as n → 0 so we cannot have ||u||∞ ≤
C||u||1 .
(iv) Let x = ni=1 ai ei and y = ni=1 bi ei . Then, x + y = ni=1 (ai + bi )ei and so
P P P
n
X n
X n
X
||x + y||1 = |ai + bi | ≤ |ai | + |bi | = ||x||1 + ||y||1 .
i=1 i=1 i=1
Pn Pn
Now let x = i=1 ai ei . Then, ||x|| ≤ i=1 |ai |||ei || ≤ M ||x||1 where M = max1≤i≤n ||ei ||.
Next we have |||x|| − ||y||| ≤ ∥x − y|| ≤ M ||x − y||1 . It follows that the map x 7→ ||x|| is
uniformly continuous on (E, || · ||1 ). Let S1 = {x ∈ E | ||x||1 = 1} be the unit sphere in
(E, || · ||1 ). Since S1 is compact, the continuous map x 7→ ||x|| has a minimum value at a point
x0 ∈ S1 . So we have ||x|| ≥ ||x0 || for all x ∈ S1 . If we set m = ||x0 ||, then m > 0 because
2
x x
otherwise x0 = 0 ∈ / S1 . Now let x ∈ E satisfy x ̸= 0. Then, ||x|| 1
∈ S1 and so ||x||1 ≥ m. It
follows that ||x|| ≥ m||x||1 . Since this inequality still holds for x = 0. We get
||x|| ≥ m||x||1 ∀x ∈ E.
Thus, the two norms are equivalent. It follows that all norms on E are equivalent.
Suppose that (E, || · ||1 ) is complete. Let (xn ) be a Cauchy sequence in (E, || · ||2 ). The
inequality ||xn − xm ||1 ≤ 1c ||xn − xm ||2 implies that (xn ) is a Cauchy sequence in (E, || · ||1 ).
Completeness of (E, || · ||1 ) implies that (xn ) converges to some x in (E, || · ||1 ). The inequality
||xn −x||2 ≤ C||xn −x||1 implies that (xn ) converges in (E, ||·||2 ). Thus, (E, ||·||2 ) is complete.
Since || · ||1 and || · ||1 play the same role, completeness of (E, || · ||2 ) implies completeness of
(E, || · ||1 ).
5. Let E and F be normed spaces and let L : E → F be a linear operator. Show that if E is finite
dimensional then L is bounded.
Solution. Let {e
P1 , . . . , en } be a basis ofP
E. Let x ∈ E. Then, x can be written in a
unique way P as x = ni=1 αi ei . Then Lx = n
i=1 αi Lei . Let M = max1≤i≤n ||Lei ||. Then
n Pn
||Lx|| ≤ M i=1 |αi |. If we set ||x||1 = i=1 |αi |, then || · ||1 is a norm equivalent to any other
norm on E (see exercise 4). Moreover ||Lx|| ≤ M ||x||1 . Since there exists a constant C such
that ||x||1 ≤ C||x||, we get ||Lx|| ≤ M C||x||. This proves the boundedness of L.
6. Let (E, || · ||E ) and (F, || · ||F ) be two normed spaces. Set
(iv) To simplify the writing we drop the subscripts. We have to prove that
3
Now by the triangle inequality for || · ||E and || · ||F we have
||x + x′ ||2 + ||y + y ′ ||2 ≤ ||x||2 + ||x′ ||2 + 2||x||||x′ || + ||y||2 + ||y ′ ||2 + 2||y||||y ′ ||.
b) We have
||(x, y)||∞ ≤ ∥(x, y)||1 ≤ 2||(x, y)||∞
and √
||(x, y)||∞ ≤ ∥(x, y)||2 ≤ 2||(x, y)||∞ .
c) Let (xn , yn ) be a Cauchy sequence in (E × F, || · ||1 ). Then, for any ε > 0 there exists n0
such that
||xn − xm || + ||yn − ym || ≤ ε
for n, m ≥ n0 . It follows that (xn ) and (yn ) are Cauchy sequences in E and F respectively.
Completeness of E and F implies that (xn ) converges to some x in E and (yn ) converges to
some y in F . It follows that
7. Let E and F be normed spaces and let L ∈ L(E, F ). Recall that the norm of L is defined by
||L|| = sup||x||≤1 ||Lx||. Show that
||Lx||
||L|| = sup ||Lx|| = sup ||Lx|| = sup = inf{C > 0 ; ||Lx|| ≤ C||x||}.
||x||=1 ||x||<1 x̸=0 ||x||
x
Set M = sup||x||=1 ||Lx|| = supx∈SE ||Lx||. Let now x ∈ BE with x ̸= 0. Then ||x|| ∈ SE .
Therefore,
x
L ≤ M.
||x||
This means that ||Lx|| ≤ M ||x|| ≤ M . Since ||L0|| = 0 ≤ M we have
||Lx|| ≤ M ∀x ∈ BE .
Let m = sup||x||<1 ||Lx||. Then, m ≤ ||L|| since {x; ||x|| < 1} ⊂ {x; ||x|| ≤ 1}. Let A = {x ∈
BE ; ||Lx|| ≤ m}. By continuity of x 7→ ||Lx||, the set A is closed. On the other hand, we have
by definition that {x; ||x|| < 1} ⊂ A. It follows that {x; ||x|| < 1} ⊂ A since A is closed. But
4
{x; ||x|| < 1} = BE . Therefore, BE ⊂ A. This means that ||Lx|| ≤ m, ∀x ∈ BE . It follows
that ||L|| ≤ m. Hence equality. Actually A = BE since by definition A ⊂ BE .
Next, note that ||Lx|| ≤ ||L||||x|| for all x ∈ E. Setting α = inf{C > 0 | ||Lx|| ≤ C||x||}, we
see that α ≤ ||L||. Conversely, let ε > 0. Then, by a property of the infimum, there exists
a positive constant Cε such that ||Lx|| ≤ Cε ||x|| ∀x ∈ E and Cε < α + ε. It follows that
||Lx|| ≤ (α + ε)||x|| ∀x ∈ E. Taking the sup over BE we get ||L|| ≤ α + ε. Since ε was
arbitrary, we get ||L|| ≤ α. Hence the last equality.
Solution. (a) Since f (x) ≤ |f (x)| for all x ∈ E, we have on the one hand
On the other hand, let x ∈ BE . Then either |f (x)| = f (x) or |f (x)| = −f (x) = f (−x). Since
−x ∈ BE we have in both cases |f (x)| ≤ supx∈BE f (x). Since x was arbitrary, it follows that
Hence the first equality. Repeating the above reasoning with SE instead of BE , proves the
second equality.
(b) Using the facts that BE is symmetric with respect to the origin and f (−x) = −f (x), we
can write
inf f (x) = − sup (−f (x)) = − sup f (x) = −||f ||.
x∈BE x∈BE x∈BE
9. Let E be a finite dimensional vector space and let {e1 , . . . , en }Pbe a basis of E. Then every
element x ∈ E can be written in a unique way in the form x = ni=1 xi ei . We set then
n n
!1/p
X X
p
||x||1 = |xi |, ||x||∞ = max |xi |, ||x||p = |xi | (1 < p < ∞).
1≤i≤n
i=1 i=1
(a) We equip E with the norm || · ||1 . Let f ∈ E ∗ and let fi = f (ei ). Show that
(b) We equip E with the norm || · ||∞ . Let f ∈ E ∗ and let fi = f (ei ). Show that
n
X
||f ||E ∗ = |fi |.
i=1
5
(c) We equip E with the norm || · ||p where 1 < p < ∞. Let f ∈ E ∗ and let fi = f (ei ). Show
that !1/p′
n
′
X
||f ||E ∗ = |fi |p
i=1
1 1
where p + p′ = 1.
n n
!1/p
p′ p′
X X
|fi | ≤ ||f || |fi | .
i=1 i=1
Therefore,
n
!1−1/p
p′
X
|fi | ≤ ||f ||.
i=1
1 1
But 1 − p = p′ . Hence the reverse inequality and therefore the equality.
10. Let E = C[0, 1]. Equip E with the norm of uniform convergence ||x|| = sup0≤t≤1 |x(t)|. Let
g ∈ E. Define the linear operator f : E → R by
Z 1
f (x) = g(t)x(t) dt.
0
6
It follows that f is bounded and ||f || ≤ ||g||1 .
Z 1
ng 2 (t)
b) We have f (xn ) = dt. Observe that ||xn || ≤ 1. Now
0 1 + n|g(t)|
ng 2 (t) |g(t)| 1
− |g(t)| = ≤
1 + n|g(t)| 1 + n|g(t)| n
ng 2 (t)
This means that the sequence of functions converges uniformly to |g(t)|. Therefore
1 + n|g(t)|
Z 1 1
ng 2 (t)
Z
converges to |g(t)| dt. This means that f (xn ) → ||g||1 . But
0 1 + n|g(t)| 0
Letting n → ∞ in the last inequality, we get ||g||1 ≤ ||f ||. Hence equality.
It follows that 1
Z 1 Z
2
cos(πt)[x0 (t) + 1] dt = cos(πt)[1 − x0 (t)] dt.
1
2
0
The integral on the left is ≤ 0 because cos(πt) ≤ 0 and x0 (t) + 1 ≥ 0, whereas the integral on
the right is ≥ 0. Therefore the two integrals are zero. Since the integrands are continuous and
are of constant sign, they are zero. Thus,
(
1 if t < 21
x0 (t) =
−1 if t > 12 .
But this contradicts the continuity of x0 . Therefore the sup is not achieved.
7
Solution. (i)
x x
x ∈ λB(a, r) ⇔ ∈ B(a, r) ⇔ || − a|| < r ⇔ ||x − λa|| < λr ⇔ x ∈ B(λa, λr).
λ λ
(ii)
x ∈ a + B(b, r) ⇔ x − a ∈ B(b, r) ⇔ ||x − a − b|| < r ⇔ x ∈ B(a + b, r).
(iii) The inclusion B(a, r) + B(b, s) ⊂ B(a + b, r + s) follows from the triangle inequality.
r s
Conversely, let x ∈ B(a + b, r + s). Set x1 = a + r+s (x − a − b) and x2 = b + r+s (x − a − b).
Then x1 ∈ B(a, r); x2 ∈ B(b, s) and x = x1 + x2 .
Remark. The open balls can be replaced by the corresponding closed balls.
12. Let A and B be two subsets of a normed space E. Show that if A is closed and B is compact
then A + B is closed.
Solution. Let x belong to the closure of A + B. Then, there exists a sequence (xn ) of A + B
that converges to x. We can write xn = an + bn where an ∈ A and bn ∈ B. Compactness of B
implies that (bn ) has a subsequence (bnk ) that converges to some b ∈ B. Since (xnk ) converges
to x, the sequence (ank ) converges to a := x − b. Since A is closed, a ∈ A. Therefore,
x = a + b ∈ A + B.
13. Let E be a normed space and let F be a subspace of E with nonempty interior. Show that
F = E.
Solution. Let a be an interior point of F . Then, there exists r > 0 such that B ′ (a, r) ⊂ F .
r ||x − a||
Now let x ∈ E. Let y = a + (x − a) so that x = a + (y − a). Observe
||x − a|| r
that ||y − a|| = r. Therefore y ∈ B ′ (a, r) ⊂ F . Since F is a subspace and a ∈ F , we have
||x − a||
x=a+ (y − a) ∈ F . Thus, E ⊂ F ⊂ E and so F = E.
r
14. Let F be a subspace of a vector space E. Let f : E → R be linear. Show that if f is bounded
from above or from below on F , then f = 0 on F .
Solution. Suppose that f is bounded from above. Then there exists a number α such that
f (x) ≤ α for all x ∈ F . Therefore for every t ∈ R and every x ∈ F we have
tf (x) ≤ α.
Suppose that f ̸= 0 on F . Then there exists x0 ∈ F such that f (x0 ) ̸= 0. We may assume
α
that f (x0 ) > 0 because if f (x0 ) < 0, then f (−x0 ) > 0. Take in particular a number t > f (x 0)
.
Then, we get α < tf (x0 ) ≤ α, a contradiction. Therefore f vanishes on F .
Now if f is bounded from below, then −f is bounded from above and so according to what we
said, −f = 0 on F . Therefore f = 0 on F .
8
◦ ◦
(c) Deduce that C = C whenever C ̸= ∅.
Solution. (a) Let x, y ∈ C and let t ∈ [0, 1]. Then, there exist a sequence (xn ) in C
converging to x and a sequence (yn ) of C converging to y. Since C is convex, (1−t)xn +tyn ∈ C.
But (1 − t)xn + tyn converges to (1 − t)x + ty. Therefore (1 − t)x + ty ∈ C.
◦
Let x, y ∈ C and let t ∈]0, 1[. Then, there exists r > 0 such that B(x, r) ⊂ C and B(y, r) ⊂ C.
Now, (1 − t)B(x, r) + tB(y, r) ⊂ (1 − t)C + tC. Since C is convex, (1 − t)C + tC ⊂ C
and so (1 − t)B(x, r) + tB(y, r) ⊂ C. By the previous exercise, (1 − t)B(x, r) + tB(y, r) =
◦
B((1 − t)x + ty, r). Therefore B((1 − t)x + ty, r) ⊂ C. This means (1 − t)x + ty ∈ C.
(b) There exists r > 0 such that B(y, r) ⊂ C. Since C is convex, (1 − t)x + tB(y, r) ⊂ C.
But it is not difficult to see that (1 − t)x + tB(y, r) = B((1 − t)x + ty, tr). It follows that
◦
(1 − t)x + ty ∈ C.
◦
(c) One inclusion is clear. Conversely, let x ∈ C and fix y0 ∈ C. By the previous question,
◦ ◦
(1 − n1 )x + n1 y0 ∈ C. But this sequence converges to x. It follows that x ∈ C. This proves that
◦ ◦
C ⊂ C and therefore C ⊂ C.
16. Let f : X → Y be a function between two metric spaces. The graph of f is the set G(f ) =
{(x, f (x)) | x ∈ X}.
Solution. (a) Let (x, y) belong to the closure of G(f ). Then, there exists a sequence
((xn , yn )) in G(f ) that converges to (x, y). Therefore xn → x and yn = f (xn ) → y. Continuity
of f implies that f (xn ) → f (x). By uniqueness of limits in metric spaces, we get y = f (x).
Therefore (x, y) ∈ G(f ).
Then G(f ) is a union of the three sets F1 = {(x, x1 ) | x > 0}; F2 = {0} and F3 = {(x, x1 ) | x < 0}.
It is not difficult to see that F1 , F2 and F3 are closed subsets of R2 . Therefore, G(f ) is closed
but f is clearly not continuous.
(c) Let x ∈ X and let xn → x in X. Compactness of Y implies that the sequence (f (xn ))
has a subsequence (f (xnk )) that converges to some y ∈ Y . Now the sequence (xnk , f (xnk ))
belongs to G(f ) and converges to (x, y). Since G(f ) is closed, (x, y) ∈ G(f ) and so y = f (x).
Therefore f (xnk ) → f (x). This means that f is continuous at x. Since x was arbitrary, f is
continuous on X.
17. (a) Show that in a normed space all open balls are homeomorphic and all closed balls are
homeomorphic.
9
(b) Let E be an infinite dimensional normed space and let K ⊂ E be compact. Show that the
interior of K is empty, i.e., K cannot contain any open ball.
(b) Suppose that K contains an open ball. Then it contains a closed ball B ′ (a, r). Being a closed
subset of a compact space, B ′ (a, r) is itself compact. It follows from the previous question that
BE is compact. But this contradicts the fact that E is infinite dimensional.
10